1
$\begingroup$

So it is well know that the Eisenstein series of weight 2 is not modular on $SL_2(\mathbb{Z})$.

In this paper of Kilford (http://uk.arxiv.org/PS_cache/math/pdf/0701/0701478v1.pdf) on page 4, he says that $E_2 \in M_2(\Gamma_0(2))$.

In fact we can obtain the following equality, $$ E_2 = \dfrac{\eta(2z)^{20}}{\eta(z)^8\eta(4z)^8} + 16\cdot \dfrac{\eta(4z)^8}{\eta(2z)^4} $$ (there's a typo in the orignal paper which is fixed here) and since the linear combination of eta-quotients on the right hand side is modular on $\Gamma_0(8)$ by a theorem of Ligozat (which can be found on page 2 of that same paper), we would have that $E_2$ is modular on $\Gamma_0(8)$ as well.

So my question is, do we have modularity for $E_2$ on $\Gamma_0(2)$ as well? I was wondering this since Kilford says that $E_2 \in M_2(\Gamma_0(2))$. If so, why? How about for $\Gamma_0(4)$?

$\endgroup$
2
  • 1
    $\begingroup$ Please note that if $f$ is a modular form for $\Gamma_0(n)$, then $f$ is a modular form for $\Gamma_0(kn)$ for all positive integers $k$. $\endgroup$
    – S. Carnahan
    Mar 27, 2012 at 8:17
  • $\begingroup$ Your $E_2$ is actually $E_{\gamma,2}$ whose $q$-series is the generating function of OEIS sequence A004011. $\endgroup$
    – Somos
    Oct 21, 2017 at 23:52

1 Answer 1

9
$\begingroup$

Kilford uses not $E_2$ but something he calls $E_{2, 2}$, which is $$E_{2}(z) - 2E_2(2z) = 1 - 24 \sum_{n \text{ odd}} \sigma_{1}(n) q^n.$$ This is a modular form of weight 2 and level $\Gamma_0(2)$. Similarly $E_{p, 2} = E_2(z) - p E_2(pz)$ is modular of level $\Gamma_0(p)$ for any $p$.

The naive Eisenstein series $$E_{2}(z) = 1 - 24 \sum_{n \in \mathbb{N}} \sigma_{1}(n) q^n$$ is not a modular form of any level.

[EDIT: My original answer contained the following statement, which is obviously wrong: "this follows from the fact that $E_2(-1/z) - z^2 E_2(z)$ is something like $6/\pi \mathrm{Im}(z)$, while for a modular form it would have to be holomorphic."

A hopefully better statement is: we have $E_2(z) - z^{-2} E_2(-1/z) = 2\pi i / z$, and if $E_2$ were modular (of some level) then $z^{-2} E_2(-1/z)$ would also be modular (of some other level) and hence their difference would be modular (for the intersection of the two level groups). But a non-constant rational function cannot be modular of any level.]

$\endgroup$
5
  • $\begingroup$ How is this possible? $E_2(z)$ is holomorphic in $z$, isn't it. So also $z^2E_2(z)$ is holomorphic and also $E_2(-1/z)$ is also holomorphic (except at $z=0$). So then the difference has to be holomorphic as well. $\endgroup$
    – wood
    Mar 27, 2012 at 10:48
  • $\begingroup$ In fact the formulas is $E_2(-1/z) - z^2 E_2(z)=-2 \pi i z$. But correcting with a non-holomorphic summand gives really modular transformation behavior. $\endgroup$
    – wood
    Mar 27, 2012 at 10:58
  • $\begingroup$ Weight-two holomorphic Eisenstein series aren't defined directly by the usual series, which doesn't converge. Rather, consider a larger family of Eisenstein series with additional complex parameter $s$, show meromorphic continuation in $s$, and evaluate at $s_o$ most likely to produce something holomorphic in $z$. However, the obstruction is non-trivial in general. For Hilbert modular forms over fields other than $\mathbb Q$ the obstruction vanishes. This game is sometimes called "Hecke summation". The obstruction tends to be in the constant term, so the trick mentioned above succeeds. $\endgroup$ Mar 27, 2012 at 17:07
  • $\begingroup$ Right, if you do the Hecke trick, the non-holomorphic part will occur naturally. I am only saying that the difference of two holomorphic functions is holomorphic. In fact the correct transformation behavior $E_2(-1/z) - z^2 E_2(z)=-2 \pi i z$ also shows that $E_2$ cannot be modular for any level. $\endgroup$
    – wood
    Mar 27, 2012 at 18:14
  • $\begingroup$ Sorry for the confusion, guys -- my original answer is obvious nonsense (I was somehow muddling the holomorphic and non-holomorphic $E_2$'s). I've replaced the bogus remark with an expansion of wood's last comment. $\endgroup$ Mar 27, 2012 at 21:03

Your Answer

By clicking “Post Your Answer”, you agree to our terms of service and acknowledge you have read our privacy policy.

Not the answer you're looking for? Browse other questions tagged or ask your own question.